LSAT and Law School Admissions Forum

Get expert LSAT preparation and law school admissions advice from PowerScore Test Preparation.

 Administrator
PowerScore Staff
  • PowerScore Staff
  • Posts: 8916
  • Joined: Feb 02, 2011
|
#23341
Complete Question Explanation

Parallel Reasoning-PR, SN. The correct answer choice is (B)

This problem is quite difficult; only 29% of all students select the correct answer choice. The principle in the stimulus is actually a conditional statement which indicates that people receiving unsolicited advice from an interested party should always be skeptical, unless their interests coincide with those of the advice giver:
  • Skeptical About Unsolicited Advice From Interested Party ..... :arrow: ..... Interests Coincide.
The question stem asks you to consider the judgment best supported by the principle above. Remember, to justify a particular conclusion in an argument utilizing a conditional statement it is necessary to satisfy the sufficient condition. But if we use the principle above, satisfying the sufficient condition of being Not Skeptical only allows to us judge whether or not Interests Coincide, and unfortunately each of the five answer choices contains a judgment which relates to following advice, not coinciding interests. Thus, to effectively use the principle above to evaluate judgments, we must use the contrapositive:
  • Interests Coincide ..... :arrow: ..... Be Skeptical About Unsolicited Advice From Interested Party.
Therefore, if we can find a situation where the interests of the two parties do not coincide, then that situation would be sufficient to justify skepticism on the part of the advisee.

Answer choice (A) is incorrect since the judgment is to "reject the offer out of hand." As noted in the principle, advisees should be "skeptical." Since the judgments do not match, answer choice (A) is incorrect.

Answer choice (B) is the correct answer choice. The interests of the two parties do not coincide and thus Ramon should be skeptical. Since the judgment in answer choice (B) essentially indicates that Ramon should be skeptical, it is correct.

In answer choice (C), Mario solicits Yvette's advice, and so the principle does not apply.

Answer choice (D) is perhaps the most complex of the four wrong answer choices, but it is incorrect for two reasons. First, Sara and Ron clearly have a coinciding interest since each wrote a chapter of the textbook, and this scenario does not conform to the sufficient condition in the principle above. Second, and perhaps more important to the answer choice ("However, even though..."), the relationship between Sara and the book editor does not necessarily serve Sara's advantage, which is one of the criteria for enacting the principle.

Answer choice (E) is incorrect since Joel's advantage is not served by the advice he offers.
 Blueballoon5%
  • Posts: 156
  • Joined: Jul 13, 2015
|
#46100
Administrator wrote: The question stem asks you to consider the judgment best supported by the principle above. Remember, to justify a particular conclusion in an argument utilizing a conditional statement it is necessary to satisfy the sufficient condition. But if we use the principle above, satisfying the sufficient condition of being Not Skeptical only allows to us judge whether or not Interests Coincide, and unfortunately each of the five answer choices contains a judgment which relates to following advice, not coinciding interests. Thus, to effectively use the principle above to evaluate judgments, we must use the contrapositive:
Could you elaborate a bit further on the meaning of the technique that is in bold (above). What does it mean for us to try and "satisfy the sufficient condition"?
 Adam Tyson
PowerScore Staff
  • PowerScore Staff
  • Posts: 5153
  • Joined: Apr 14, 2011
|
#49904
That just means you need to show that the sufficient condition occurred, blueballoon. If you are given a conditional statement - if A, then B - and the author then concludes that B must have occurred, you can justify the conclusion only by showing that the sufficient condition occurred, or that the sufficient condition "was satisfied." One you have the sufficient condition satisfied, or occurring, then you know for sure that the necessary condition must also occur.
 yournoona
  • Posts: 18
  • Joined: Mar 13, 2020
|
#74412
Administrator wrote: Complete Question Explanation

Answer choice (A) is incorrect since the judgment is to "reject the offer out of hand." As noted in the principle, advisees should be "skeptical." Since the judgments do not match, answer choice (A) is incorrect.

Answer choice (B) is the correct answer choice. The interests of the two parties do not coincide and thus Ramon should be skeptical. Since the judgment in answer choice (B) essentially indicates that Ramon should be skeptical, it is correct.
I did get this answer by elimination, but i still don't understand as to why B is correct.
According to the conditional reasoning it should be,

No coincidence of interests = Skeptical

I get till the part where there is no coincidence of interests: Shopkeeper wants his profits and Ramon wants to save his money, however the second part doesn't show any Skepticism from Ramon's side.......It only gives us that Ramon should not reject the advice of the Shopkeeper. I don't understand as to how this constitutes as Skepticism. It is mere rejection and if anything, it is more firm in the belief of not accepting which is hardly skeptical.
 Paul Marsh
PowerScore Staff
  • PowerScore Staff
  • Posts: 290
  • Joined: Oct 15, 2019
|
#74427
Hi yournoona! Nice job writing out that conditional correctly. However, based on what you wrote, I think you may have mis-read the end of Answer Choice (B). (B) concludes that "Ramon should not reject the least expensive model on the salesperson's advice alone". In other words, (B) isn't saying that Ramon shouldn't reject the salesperson's advice; rather, it's saying that he shouldn't reject the least expensive fridge. Not rejecting the least expensive fridge would be against the salesperson's advice, thereby showing that Ramon is somewhat skeptical of that advice. Hope that helps!
 yournoona
  • Posts: 18
  • Joined: Mar 13, 2020
|
#74541
Yes, i got it. Thankyou so much for your effort.
User avatar
 abby1285
  • Posts: 11
  • Joined: Mar 20, 2021
|
#87207
Hi! I was tripped on B because of the phrase "on the basis of her personal experience" I thought this implied that they did have aligning interests- could someone explain? Thanks!
 Robert Carroll
PowerScore Staff
  • PowerScore Staff
  • Posts: 1787
  • Joined: Dec 06, 2013
|
#87220
Abby,

Note that the salesperson gives advice based on her (the salesperson's) personal experience. So ask - is what's valid for one person's experience valid for all? Without further info, I wouldn't say so. We'd have to have info either about a matching of their personal experiences in the relevant way, or else some info that the salesperson's experience would per se be relevant to Ramon's. For the former, I think it would have to say something like "Ramon's needs are very much like those of the salesperson, who found herself that the cheapest fridge was a disappointment". For the latter, maybe "In her experience, the cheapest fridge requires expensive repairs" or something where we could probably assume Ramon also wants to avoid expensive repairs.

I'm not saying my additional info is perfect or that it would definitely change my answer, but without some extra info like that provided, we don't know if the interests line up. Maybe Ramon just needs to keep drinks cold, and the salesperson needed to keep leftovers cold.

Keep in mind that even in those situations, the fact that the salesperson gets a commission, and, of course, Ramon doesn't, already introduces a potential lack of coincidence of interests.

Robert Carroll

Get the most out of your LSAT Prep Plus subscription.

Analyze and track your performance with our Testing and Analytics Package.